You are on page 1of 47

Root, Prefix or Suffix Meaning Examples

a, ac, ad, af, ag, al, an,


ap, as, at
to, toward, near, in
addition to, by
aside, accompany, adjust, aggression,
allocate, annihilate, affix, associate, attend,
adverb
a-, an- not, without
apolitical, atheist, anarchy, anonymous,
apathy, aphasia, anemia
ab, abs away from, off absolve, abrupt, absent
-able, -ible Adjective: worth, ability solvable, incredible
acer, acid, acri bitter, sour, sharp acerbic, acidity, acrid, acrimony
act, ag do, act, drive active, react, agent, active, agitate
acu sharp acute, acupuncture, accurate
-acy, -cy Noun: state or quality
privacy, infancy, adequacy, intimacy,
supremacy
-ade act, product, sweet drink blockade, lemonade
aer, aero air, atmosphere, aviation aerial, aerosol, aerodrome
ag, agi, ig, act do, move, go
agent, agenda, agitate, navigate,
ambiguous, action
-age
Noun: activity, or result of
action courage, suffrage, shrinkage, tonnage
agri, agro pertaining to fields or soil agriculture, agroindustry
-al
Noun: action, result of
action referral, disavowal, disposal, festival
-al, -ial, -ical Adjective: quality, relation structural, territorial, categorical
alb, albo white, without pigment albino, albite
ali, allo, alter other
alias, alibi, alien, alloy, alter, alter ego,
altruism
alt high, deep altimeter, altitude
am, ami, amor love, like, liking amorous, amiable, amicable, enamoured
ambi both ambidextrous
ambul to walk
ambulatory, amble, ambulance,
somnambulist
-an Noun: person artisan, guardian, historian, magician
ana, ano up, back, again, anew anode, anagram, anagenetic
-ance, -ence
Noun: action, state,
quality or process
resistance, independence, extravagance,
fraudulence
-ancy, -ency
Noun: state, quality or
capacity vacancy, agency, truancy, latency
andr, andro
male, characteristics of
men androcentric, android
ang angular angle
anim mind, life, spirit, anger animal, animate, animosity
ann, annu, enni yearly
annual, annual, annuity, anniversary,
perrenial
-ant, -ent
Noun: an agent,
something that performs
the action disinfectant, dependent, fragrant
-ant, -ent, -ient
Adjective: kind of agent,
indication important, dependent, convenient
ante before
anterior, anteroom, antebellum, antedate,
antecedent antediluvian
anthrop man anthropology, misanthrope, philanthropy
anti, ant against, opposite
antisocial, antiseptic, antithesis, antibody,
antinomies, antifreeze, antipathy
anti, antico old antique, antiquated, antiquity
apo, ap, aph
away from, detached,
formed apology, apocalypse, aphagia
aqu water aqueous
-ar, -ary
Adjective: resembling,
related to spectacular, unitary
arch chief, first, rule
archangel, architect, archaic, monarchy,
matriarchy, patriarchy, Archeozoic era
-ard, -art Noun: characterized braggart, drunkard, wizard
aster, astr star
aster, asterisk, asteroid, astronomy,
astronaut
-ate Noun: state, office, fuction candidate, electorate, delegate
-ate Verb: cause to be graduate, ameliorate, amputate, colligate
-ate Adjective: kind of state inviolate
-ation
Noun: action, resulting
state specialization, aggravation, alternation
auc, aug, aut to originate, to increase augment , author, augment, auction
aud, audi, aur, aus to hear, listen
audience, audio, audible, auditorium,
audiovisual, audition, auricular, ausculate
aug, auc increase augur, augment, auction
aut, auto self
automobile, automatic, automotive,
autograph, autonomous, autoimmune
bar weight, pressure barometer
be
on, around, over, about,
excessively, make,
cause, name, affect berate, bedeck, bespeak, belittle, beleaguer
belli war rebellion, belligerent, casus belli, bellicose
bene good, well, gentle
benefactor, beneficial, benevolent,
benediction, beneficiary, benefit
bi, bine two biped, bifurcate, biweekly, bivalve, biannual
bibl, bibli, biblio book bibliophile, bibliography, Bible
bio, bi life
biography, biology, biometricsm biome,
biosphere
brev short abbreviate, brevity, brief
cad, cap, cas, ceiv, cept,
capt, cid, cip to take, to seize, to hold
receive, deceive, capable, capacious,
captive, accident, capture, occasion,
concept, intercept, forceps, except,
reciprocate
cad, cas to fall cadaver, cadence, cascade
-cade procession motorcade
calor heat calorie, caloric, calorimeter
capit, capt head decapitate, capital, captain, caption
carn flesh carnivorous, incarnate, reincarnation, carnal
cat, cata, cath down, with catalogue, category, catheter
caus, caut burn, heat caustic, cauldron, cauterize
cause, cuse, cus cause, motive because, excuse, accusation
ceas, ced, cede, ceed,
cess
to go, to yield, move, go,
surrender
succeed, proceed, precede, recede,
secession, exceed, succession
cent hundred centennial, century, centipede
centr, centri center
eccentricity, centrifugal, concentric,
eccentric
chrom color
chrome, chromosome, polychrome,
chromatic
chron time
chronology, chronic, chronicle chronometer,
anachronism, synchronize
cide, cis, cise to kill, to cut, cut down
fratricide, homicide, incision, incision,
circumcision, scissors
circum around
circumnavigate, circumflex, circumstance,
circumcision, circumference, circumorbital,
circumlocution, circumvent, circumscribe,
circulatory
cit call, start incite, citation, cite
civ citizen civic, civil, civilian, civilization
clam, claim cry out
exclamation, clamor, proclamation,
reclamation, acclaim
clin lean, bend decline, aclinic, inclination
clud, clus claus to close, shut
include, exclude, clause, claustrophobia,
enclose, exclusive, reclusive, conclude
co, cog, col, coll, con,
com, cor with, together
cohesiveness, cognate, collaborate,
convene, commitment, compress,
contemporary, converge, compact,
confluence, convenient, concatenate,
conjoin, combine, correct
cogn, gnos to know
recognize, cognizant, diagnose, agnostic,
incognito, prognosis
com, con fully
complete, compel, conscious, condense,
confess, confirm
contr, contra, counter against, opposite
contradict, counteract, contravene, contrary,
counterspy, contrapuntal
cord, cor, cardi heart
cordial, concord, discord, courage,
encourage
corp body
corporation, corporal punishment, corpse,
corpulent, corpus luteum
cort correct escort, cortage
cosm universe, world
cosmos, microcosm, cosmopolitan,
cosmonaut
cour, cur, curr, curs run, course occur, excursion, discourse, courier, course
crat, cracy rule autocrat, aristocrat, theocracy, technocracy
cre, cresc, cret, crease grow create, crescent, accretion, increase
crea create creature, recreation, creation
cred believe
creed, credo, credence, credit, credulous,
incredulous, incredible
cresc, cret, crease, cru rise, grow
crescendo, concrete, increase, decrease,
accrue
crit separate, choose critical, criterion, hypocrite
cur, curs run
current, concurrent, concur, incur, recur,
occur, courier, precursor, cursive
cura care curator, curative, manicure
cycl, cyclo wheel, circle, circular Cyclops, unicycle, bicycle, cyclone, cyclic
de-
from, down, away, to do
the opposite, reverse,
against
detach, deploy, derange, decrease,
deodorize, devoid, deflate, degenerate
dec, deca ten, ten times
decimal, decade, decalogue, decimate,
decathlon
dec, dign suitable decent decorate dignity
dei, div God divinity, divine, deity, divination, deify
dem, demo
people, populace,
population
democracy, demography, demagogue,
epidemic
dent, dont tooth dental, denture, orthodontist, periodontal
derm skin, covering
hypodermic, dermatology, epidermis,
taxidermy
di-, dy- two, twice, double divide, diverge, diglycerides
dia through, across, between
diameter, diagonal, dialogue dialect,
dialectic, diagnosis, diachronic
dic, dict, dit say, speak
dictation, dictionary, dictate, dictator,
Dictaphone, edict, predict, verdict,
contradict, benediction
dis, dif
not, opposite of, reverse,
separate, deprive of,
away
dismiss, differ, disallow, disperse, dissuade,
divide, disconnect, disproportion, disrespect,
distemper, disarray
dit give credit, audit
doc, doct teach, prove
docile, doctor, doctrine, document, dogma,
indoctrinate
domin
master, that which is
under control dominate, dominion, predominant, domain
don give donate, condone
dorm sleep dormant, dormitory
dox thought, opinion, praise orthodox, heterodox, paradox, doxology
-drome run, step syndrome, aerodrome, velodrome
duc, duct to lead, pull
produce, abduct, product, transducer,
viaduct, aqueduct, induct, deduct, reduce,
induce
dura hard, lasting durable, duration, endure
dynam power dynamo, dynamic, dynamite, hydrodynamics
dys-
bad, abnormal, difficult,
impaired, unfavorable dysfunctional, dyslexia, dyspathy
e-
not, missing, out, fully,
away, computer network
related
emit, embed, eternal,ether, erase, email, e-
tailer
ec- out of, outside
echo, eclipse, eclectic, ecesis, ecstasy,
exzema
eco-
household, environment,
relating to ecology or
economy
ecology, economize, ecospheres,
ecomanagement
ecto- outside, external ectomorph, ectoderm, ectoplasm
-ed Verb: past tense dressed, faded, patted, closed, introduced
-ed
Adjective: having the
quality or characteristics
of winged, moneyed, dogged, tiered
-en Verb: to cause to become lengthen, moisten, sharpen
-en Adjective: material golden, woolen, silken
en-, em-
put into, make, provide
with, surround with
enamor, embolden, enslave, empower,
entangle
-ence, -ency
Noun: action or process,
quality or state
reference, emergency, dependence,
eminence, latency
end- inside, within
endorse, endocardial, endergonic,
endoskeleton, endoscope, endogenous
epi-
upon, close to, over, after,
altered
epicenter, epicarp, epilogue, epigone,
epidiorite
equi- equal
equidistant, equilateral, equilibrium, equinox,
equation, equator
-er, -ier Adjective: comparative better, brighter, sooner, hotter, happier
-er, -or
Noun: person or thing that
does something
flyer, reporter, player, member, fryer,
collector, concentrator
-er, -or Verb: action ponder, dishonor, clamor
erg work, effect
energy, erg, allergy, ergometer, ergograph,
ergophobia
-ery
collective qualities, art,
practice, trade, collection,
state, condition snobbery, bakery, geenery, gallery, slavery
-es, -ies
Noun: plural of most
nouns ending in -ch, -s, -
sh, -o and -z and some in -
f and -y passes, glasses, ladies, heroes
-es, -ies
Verb: third person
singular present indicative
of verbs that end in -ch, -
s, -sh, - and some in -y blesses, hushes, fizzes, defies
-ess female actress, goddess, poetess
-est, -iest
Adjective or Adverb:
superlative latest, strongest, luckiest, lyingest
ev-, et- time, age medieval, eternal
ex-
out of, away from,
lacking, former
exit, exhale, exclusive, exceed, explosion,
ex-mayor
exter-, extra-, extro- outside of, beyond
external, extrinsic, extraordinary,
extrapolate, extraneous, extrovert
fa, fess speak
fable, fabulous, fame, famous, confess,
profess
fac, fact, fec, fect, fic, fas,
fea do, make
difficult, fashion, feasible, feature, factory,
fact, effect, manufacture, amplification,
confection
fall, fals deceive fallacy, falsify, fallacious
femto quadrillionth femtosecond
fer bear, carry
ferry, coniferous, fertile, defer, infer, refer,
transfer
fic, feign, fain, fit, feat shape, make, fashion fiction, faint, feign
fid belief, faith confide, diffident, fidelity
fid, fide, feder faith, trust
confidante, fidelity, confident, infidelity,
infidel, federal, confederacy, semper fi
fig shape, form figurem, effigy, figure, figment
fila, fili thread filigree, filament, filter, filet, filibuster
fin end, ended, finished
final, finite, finish, confine, fine, refine,
define, finale
fix repair, attach fix, fixation, fixture, affix, prefix, suffix
flex, flect bend
flex, reflex, flexible, flexor, inflexibility,
reflect, deflect,circumflex
flict strike affliction, conflict, inflict
flu, fluc, fluv, flux flow
influence, fluid, flue, flush, fluently, fluctuate,
reflux, influx
-fold
Adverb: in a manner of,
marked by fourfold
for, fore before forecast, fortune, foresee
forc, fort strength, strong
effort, fort, forte, fortifiable, fortify, forte,
fortitude
form shape, resemble
form, format, conform, formulate, perform,
formal, formula
fract, frag, frai break
fracture, infraction, fragile, fraction, refract,
frail
fuge flee subterfuge, refuge, centrifuge
-ful
Noun: an amount or
quanity that fills mouthful
-ful
Adjective: having, giving,
marked by fanciful
fuse pour confuse, transfuse
-fy make, form into falsify, dandify
gam marriage bigamy, monogamy, polygamy
gastr, gastro stomach gastric, gastronomic, gastritis, gastropod
gen kind generous
gen birth, race, produce
genesis, genetics, eugenics, genealogy,
generate, genetic, antigen, pathogen
geo earth geometry, geography, geocentric, geology
germ vital part germination, germ, germane
gest carry, bear congest, gestation
giga billion gigabyte, gigaflop
gin careful gingerly
gloss, glot tongue glossary, polyglot, epiglottis
glu, glo lump, bond, glue glue, agglutinate, conglomerate
gor
to gather, to bring
together category, categorize
grad, gress, gree
to gather, to bring
together, step, go
grade, degree, progress, gradual, graduate,
egress
graph, gram, graf write, written, draw
graph, graphic, autograph, photography,
graphite, telegram, polygraph, grammar,
biography, lithograph, graphic
grat pleasing congratulate, gratuity, grateful, ingrate
grav heavy, weighty grave, gravity, aggravate, gravitate
greg herd
gregarious, congregation, segregate,
gregarian
hale, heal make whole, sound inhale, exhale, heal, healthy, healthiness
helio sun heliograph, heliotrope, heliocentric
hema, hemo blood
hemorrhage, hemoglobin, hemophilia,
hemostat
her, here, hes stick
adhere, cohere, cohesion, inherent,
hereditary, hesitate
hetero other, different
heterodox, heterogeneous, heterosexual,
heterodyne
hex, ses, sex six hexagon, hexameter, sestet, sextuplets
homo same
homogenize, homosexual, homonym,
homophone
hum, human earth, ground, man humus, exhume, humane
hydr, hydra, hydro water
dehydrate, hydrant, hydraulic, hydraulics,
hydrogen, hydrophobia
hyper over, above
hyperactive, hypertensive, hyperbolic,
hypersensitive, hyperventilate, hyperkinetic
hypn sleep hypnosis, hypnotherapy
-ia Noun: names, diseases phobia
-ian, an
Noun: related to, one that
is pedestrian, human
-iatry Noun: art of healing psychiatry
-ic Adjective: quality, relation generic
-ic, ics
Noun: related to the arts
and sciences arithmetic, economics
-ice Noun: act malice
-ify Verb: cause specify
ignis fire ignite, igneous, ignition
-ile
Adjective: having the
qualities of projectile
in, im into, on, near, towards instead, import
in, im, il, ir not
illegible, irresolute, inaction, inviolate,
innocuous, intractable, innocent,
impregnable, impossible, imposter
infra beneath infrared, infrastructure
-ing
Noun: material made for,
activity, result of an
activity flooring, swimming, building
-ing Verb: present participle depicting
-ing Adjective: activity cohering
inter between, among
international, intercept, interject,
intermission, internal, intermittent,
intra
within, during, between
layers, underneath intramural, intranet, intranatal
intro into, within, inward interoffice, introvert, introspection, introduce
-ion Noun: condition or action abduction
-ish
Adjective: having the
character of newish
-ism
Noun: doctrine, belief,
action or conduct formalism
-ist Noun: person or member podiatrist
-ite Noun: state or quality graphite
-ity, ty Noun: state or quality lucidity, novelty
-ive Noun: condition native
-ive, -ative, -itive
Adjective: having the
quality of festive, cooperative, sensitive
-ize Verb: cause fantasize
jac, ject throw
reject, eject, project, trajectory, interject,
dejected, inject, ejaculate, adjacent
join, junct join
adjoining, enjoin, juncture, conjunction,
injunction, conjunction
judice judge prejudice
jug, junct, just to join junction, adjust, conjugal
juven young juvenile, rejuvenate
labor work laborious, belabor
lau, lav, lot, lut wash launder, lavatory, lotion, ablution, dilute
lect, leg, lig
choose, gather, select,
read collect, legible, eligible
leg law legal, legislate, legislature, legitimize
-less
Adjective: without,
missing motiveless
levi light alleviate, levitate, levity
lex, leag, leg law legal, college, league
liber, liver free liberty, liberal, liberalize, deliverance
lide strike collide, nuclide
liter letters
literary, literature, literal, alliteration,
obliterate
loc, loco place, area location, locally, locality, allocate, locomotion
log, logo, ology
word, study, say, speech,
reason, study catalog, prologue, dialogue, zoology, logo
loqu, locut talk, speak
eloquent, loquacious, colloquial,
circumlocution
luc, lum, lun, lus, lust light
translucent, luminary, luster, luna, illuminate,
illustrate
lude play prelude
-ly Adverb: in the manner of fluently
macr-, macer lean emaciated, meager
magn great
magnify, magnificent, magnanimous,
magnate, magnitude, magnum
main strength, foremost mainstream, mainsail, domain, remain
mal bad, badly
malformation, maladjusted, dismal, malady,
malcontent,malfunction, malfeasance,
maleficent
man, manu hand, make, do
manual, manage, manufacture, manacle,
manicure, manifest, maneuver, emancipate,
management
mand command mandatory, remand, mandate
mania madness mania, maniac, kleptomania, pyromania
mar, mari, mer sea, pool marine, marsh, maritime, mermaid
matri mother matrimony, maternal, matriarchate, matron
medi
half, middle, between,
halfway
mediate, medieval, Mediterranean,
mediocre, medium
mega great, million
megaphone, megaton, megaflop,
megalomaniac, megabyte, megalopolis
mem recall, remember
memo, commemoration, memento, memoir,
memorable
ment mind mental, mention
-ment Noun: condition or result document
meso middle mesomorph, mesoamerica, mesosphere
meta beyond, change
metaphor, metamorphosis, metabolism,
metahistorical, metainformation
meter measure
meter, voltammeter, barometer,
thermometer
metr admeasure, apportion metrics, asymmetric, parametric, telemetry
micro small, millionth
microscope, microfilm, microcard,
microwave, micrometer, microvolt
migra wander migrate, emigrant, immigrate
mill, kilo thousand millennium, kilobyte, kiloton
milli thousandth millisecond, milligram, millivolt
min little, small minute, minor, minuscule
mis wrong, bad, badly
misconduct, misinform, misinterpret,
mispronounce, misnomer, mistake,
misogynist
mit, miss send
emit, remit, submit, admit, commit, permit,
transmit, omit, intermittent, mission, missile
mob, mov, mot move motion, remove, mobile, motor
mon warn, remind
monument, admonition, monitor,
premonition
mono one
monopoly, monotype, monologue,
mononucleosis, monorail, monotheist,
mor, mort mortal, death
mortal, immortal, mortality, mortician,
mortuary
morph shape, form
amorphous, dimorphic, metamorphosis,
morphology, polymorphic, morpheme,
amorphous
multi many, much
multifold, multilingual, multiped, multiply,
multitude, multipurpose, multinational
nano billionth nanosecond, nanobucks
nasc, nat, gnant, nai to be born nascent, native, pregnant, naive
nat, nasc to be from, to spring forth innate, natal, native, renaissance
neo new
Neolithic, nuveau riche, neologism,
neophyte, neonate
-ness
Noun: state, condition,
quality kindness
neur nerve
neuritis, neuropathic, neurologist, neural,
neurotic
nom law, order
autonomy, astronomy, gastronomy,
economy
nom, nym name nominate, synonym
nomen, nomin name nomenclature, nominate, ignominious
non nine nonagon
non not
nonferrous, nonsense, nonabrasive,
nondescript
nov new novel, renovate, novice, nova, innovate
nox, noc night nocturnal, equinox, noctilucent
numer number
numeral, numeration, enumerate,
innumerable
numisma coin numismatics
ob, oc, of, op
toward, against, in the
way oppose, occur, offer, obtain
oct eight octopus, octagon, octogenarian, octave
oligo few, little
Oligocene, oligosaccharide, oligotrophic,
oligarchy
omni all, every
omnipotent, omniscient, omnipresent,
omnivorous
onym name
anonymous, pseudonym, antonym,
synonym
oper work operate, cooperate, opus
-or Noun: condition or activity valor, honor, humor, minor
ortho straight, correct
orthodox, orthodontist, orthopedic,
unorthodox
-ory
Noun: place for, serves
for territory, rectory
-ous, -eous, -ose, -ious
Adjective: having the
quality of, relating to adventurous, courageous, verbose, fractious
over excessive, above overwork, overall, overwork
pac peace pacifist, pacify, pacific ocean
pair, pare arrange, assemblage, two repair, impair, compare, prepare
paleo old
Paleozoic, Paleolithic, paleomagnetism,
paleopsychology
pan all
Pan-American, pan-African, panacea,
pandemonium (place of all the demons),
para beside
paradox, paraprofessional, paramedic,
paraphrase, parachute
pat, pass, path feel, suffer patient, passion, sympathy, pathology
pater, patr father paternity, patriarch, patriot, patron, patronize
path, pathy feeling, suffering
pathos, sympathy, antipathy, apathy,
telepathy
ped, pod foot
pedal, impede, pedestrian, centipede, tripod,
podiatry, antipode, podium
pedo child orthopedic, pedagogue, pediatrics
pel, puls drive, push, urge
compel, dispel, expel, repel, propel, pulse,
impulse, pulsate, compulsory, expulsion,
repulsive
pend, pens, pond hang, weigh
pendant, pendulum, suspend, appendage,
pensive, append
per through, intensive
persecute, permit, perspire, perforate,
persuade
peri around periscope, perimeter, perigee, periodontal
phage eat macrophage, bacteriophage
phan, phas, phen, fan,
phant, fant show, make visible phantom, fantasy
phe speak
blaspheme, cipher, phenomenon,
philosopher
phil love
philosopher, philanthropy, philharmonic,
bibliophile
phlegma inflammation phlegm, phlegmatic
phobia, phobos fear
phobia, claustrophobia, acrophobia,
aquaphobia, ergophobia, homophobia
phon sound
telephone, phonics, phonograph, phonetic,
homophone, microphone, symphony,
euphonious
phot, photo light
photograph, photoelectric, photogenic,
photosynthesis, photon
pico trillionth picofarad, picocurie, picovolt
pict paint, show, draw picture, depict
plac, plais please placid, placebo, placate, complacent
pli, ply fold reply, implicate, ply
plore cry out, wail implore, exploration, deploring
plu, plur, plus more plural, pluralist, plus
pneuma, pneumon breath pneumatic, pneumonia,
pod foot, feet podiatry, tripod
poli city
metropolis, police, politics, Indianapolis,
megalopolis, acropolis
poly many
polytheist, polygon, polygamy,
polymorphous
pon, pos, pound place, put
postpone, component, opponent, proponent,
expose, impose, deposit, posture, position,
expound, impound
pop people population, populous, popular
port carry
porter, portable, transport, report, export,
import, support, transportation
portion part, share portion, proportion
post after, behind postpone, postdate
pot power potential, potentate, impotent
pre, pur before precede
prehendere seize, grasp
apprehend, comprehend, comprehensive,
prehensile
prin, prim, prime first
primacy, prima donna, primitive, primary,
primal, primeval, prince, principal
pro for, foward propel
proto first
prototype, protocol, protagonist, protozoan,
Proterozoic, protoindustrial
psych mind, soul psyche, psychiatry, psychology, psychosis
punct point, dot
punctual, punctuation, puncture,
acupuncture, punctuation
pute think dispute, computer
quat, quad four quadrangle, quadruplets
quint, penta five
quintet, quintuplets, pentagon, pentane,
pentameter
quip ship equip, equipment
quir, quis, quest, quer seek, ask query, inquire, exquisite, quest
re back, again report, realign, retract, revise, regain
reg, recti straighten
regiment, regular, rectify, correct, direct,
rectangle
retro backwards
retrorocket, retrospect, retrogression,
retroactive
ri, ridi, risi laughter deride, ridicule, ridiculous, derision, risible
rog, roga ask prerogative, interrogation, derogatory
rupt break rupture, interrupt, abrupt, disrupt, ruptible
sacr, sanc, secr sacred
sacred, sacrosanct, sanction, consecrate,
desecrate
salv, salu safe, healthy salvation, salvage, salutation
sanct holy
sanctify, sanctuary, sanction,
sanctimonious, sacrosanct
sat, satis enough satient, saturate, satisfy
sci, scio, scientia know science, conscious, omniscient, cognocienti
scope see, watch
telescope, microscope, kaleidoscope,
periscope, stethoscope
scrib, script write
scribe, scribble, inscribe, describe,
subscribe, prescribe, manuscript
se apart, move away from secede
sect, sec cut intersect, transect, dissect, secant, section
sed, sess, sid sit
sediment, session, obsession, possess,
preside, president, reside, subside
semi half, partial
semifinal, semiconscious, semiannual,
semimonthly, semicircle
sen, scen old, grow old
senior, senator, senile, senescence,
evanescent
sent, sens feel, think
sentiment, consent, resent, dissent,
sentimental, sense, sensation, sensitive,
sensory, dissension
sept seven septet, septennial
sequ, secu, sue follow
sequence, consequence, sequel,
subsequent, prosecute, consecutive,
second, ensue, pursue
serv save, serve, keep
servant, service, subservient, servitude,
preserve, conserve, reservation, deserve,
conservation, observe
-ship Noun: status, condition relationship, friendship
sign, signi sign, mark, seal signal, signature, design, insignia, significant
simil, simul like, resembling
similar, assimilate, simulate, simulacrum,
simultaneous
sist, sta, stit
stand, withstand, make
up
assist, insist, persist, circumstance, stamina,
status, state, static, stable, stationary,
substitute
soci to join, companions sociable, society
sol, solus alone
solo, soliloquy, solitaire, solitude, solitary,
isolate
solv, solu, solut loosen, explain
solvent, solve, absolve, resolve, soluble,
solution, resolution, resolute, dissolute,
absolution
somn sleep insomnia, somnambulist
soph wise
sophomore (wise fool), philosophy,
sophisticated
spec, spect, spi, spic look, see
specimen, specific, spectator, spectacle,
aspect, speculate, inspect, respect,
prospect, retrospective, introspective,
expect, conspicuous
sper render favorable prosper
sphere ball, sphere sphere, stratosphere, hemisphere, spheroid
spir breath
spirit, conspire, inspire, aspire, expire,
perspire, respiration
stand, stant, stab, stat,
stan, sti, sta, st, stead stand stature, establish, stance
-ster person mobster, monster
strain, strict, string, stige bind, pull, draw tight
stringent, strict, restrict, constrict, restrain,
boa constrictor
stru, struct, stroy, stry build
construe, structure, construct, instruct,
obstruct, destruction, destroy, industry,
ministry
sub, suc, suf, sup, sur,
sus
under, below, from,
secretly, instead of
sustain, survive, support, suffice, succeed,
submerge, submarine, substandard,
subnormal, subvert
sume, sump take, use, waste consume, assume, sump, presumption
super, supra over, above
superior, suprarenal, superscript,
supernatural, superimpose, supercede
syn, sym
together, at the same
time
sympathy, synthesis, synchronous,
syndicate
tact, tang, tag, tig, ting touch
tactile, contact, intact, intangible, tangible,
contagious, contiguous, contingent
tain, ten, tent, tin hold, keep, have retain, continue, content, tenacious
tect, teg cover detect, protect, tegular, tegument
tele distance, far, from afar
telephone, telegraph, telegram, telescope,
television, telephoto, telecast, telepathy,
telepathy
tem, tempo time
tempo, temporary, extemporaneously,
contemporary, pro tem, temporal
ten, tin, tain hold
tenacious, tenant, tenure, untenable,
detention, retentive, content, pertinent,
continent, obstinate, contain, abstain,
pertain, detain
tend, tent, tens stretch, strain
tendency, extend, intend, contend, pretend,
superintend, tender, extent, tension,
pretense
tera trillion terabyte, teraflop
term end, boundary, limit exterminate, terminal
terr, terra earth terrain, terrarium, territory, terrestrial
test to bear witness testament, detest, testimony, attest, testify
the, theo God, a god monotheism, polytheism, atheism, theology
therm heat
thermometer, theorem, thermal, thermos
bottle, thermostat, hypothermia
thesis, thet place, put antithesis, hypothesis, synthesis, epithet
tire draw, pull attire, retire, entire
tom cut
atom (not cutable), appendectomy,
tonsillectomy, dichotomy, anatomy
tor, tors, tort twist
torture, retort, extort, distort, contort, torsion,
tortuous, torturous
tox poison toxic, intoxicate, antitoxin
tract, tra, trai, treat drag, draw, pull
attract, tractor, traction, extract, retract,
protract, detract, subtract, contract,
intractable
trans across, beyond, change
transform, transoceanic, transmit,
transportation, transducer
tri three tripod, triangle, trinity, trilateral
trib pay, bestow
tribute, contribute, attribute, retribution,
tributary
tribute give contribute, distribute, tributary
turbo disturb turbulent, disturb, turbid, turmoil
typ print
type, prototype, typical, typography,
typewriter, typology, typify
ultima last ultimate, ultimatum
umber, umbraticum shadow
umbra, penumbra, (take) umbrage,
adumbrate
un not, against, opposite unceasing, unequal
uni one
uniform, unilateral, universal, unity,
unanimous, unite, unison, unicorn
-ure
Noun: act, condition,
process, function exposure, conjecture, measure
vac empty
vacate, vacuum, evacuate, vacation, vacant,
vacuous
vade go evade, invader
vale, vali, valu strength, worth
equivalent, valiant, validity, evaluate, value,
valor
veh, vect to carry vector, vehicle, convection, vehement
ven, vent come
convene, intervene, venue, convenient,
avenue, circumvent, invent, convent,
venture, event, advent, prevent
ver, veri true
very, aver, verdict, verity, verify,
verisimilitude
verb, verv word verify, veracity, verbalize, verve
vert, vers turn, change
convert, revert, advertise, versatile, vertigo,
invert, reversion, extravert, introvert,
diversion, introvert, convertible, reverse,
controversy
vi way viable, vibrate, vibrant
vic, vicis change, substitute vicarious, vicar, vicissitude
vict, vinc conquer victor, evict, convict, convince, invincible
vid, vis see
video, evident, provide, providence, visible,
revise, supervise, vista, visit, vision, review,
indivisible
viv, vita, vivi alive, life
revive, survive, vivid, vivacious, vitality,
vivisection, vital, vitamins, revitalize
voc, voke call
vocation, avocation, convocation, invocation,
evoke, provoke, revoke, advocate,
provocative, vocal
vol will malevolent, benevolent, volunteer, volition
volcan fire volcano, vulcanize, Vulcan
volv, volt, vol turn about, roll
revolve, voluble, voluminous, convolution,
revolt, evolution
vor eat greedily
voracious, carnivorous, herbivorous,
omnivorous, devour
-ward
Adverb: in a direction or
manner homeward
-wise
Adverb: in the manner of,
with regard to timewise, clockwise, bitwise
with against withhold, without, withdraw, forthwith
-y
Noun: state, condition,
result of an activity society, victory
-y
Adjective: marked by,
having hungry, angry, smeary, teary
zo animal
zoo (zoological garden), zoology, zodiac,
protozoan
1. There has been a sharp increase in the subscription prices of many accounting school text books in the past five years. Many publishers ascribe the necessity for these increases to the easy availability of electronic books, which enable people to electronically copy the books they want rather than buying the printed text.
Which of the following, if true, would make this explanation more plausible?
A. The great majority of student texts have a massive backlog awaiting publication.
B. Over the past five years there has been a substantial decline in the number of accounting school students, while electronic books have remained fairly stable.
C. In the five years immediately preceding the price surge, there was a substantial decline in the number of accounting school students requiring text books, while electronic book subscriptions remained fairly stable.
D. Many electronic publishers have recently begun cutting back on subscriptions of accounting school text books.
E. In almost every publishing company, there has been an increase in the number of accounting school texts available in the past few years.
2. A pesticide producing company states that their unused pesticide that gets dumped does not pose a threat to the aquatic life in the surrounding area. If this is correct, then why have local fish been dying in this region? Due to the fact that the pesticide company is not located in a highly fish-populated area, they implicitly admit that the pesticides they produce are relatively dangerous to the nearby aquatic life.
Of the following statements listed below, which one would be most likely to weaken the argument of the author if it were true?
A. The possibility of pesticides filtering into the local water region was underestimated in the past.
B. Funds for environmental company cleanup, which concern waste dumps that are poorly run, are reserved for rural regions only.
C. It would be pointless to locate chemical dumps where they would be most harmful, unless they can be proven 100-percent safe.
D. Dumps that are located in areas without large fish populations have fewer government interventions and are also less expensive.
E. City people are most probable to sue the company if the dumps cause them health problems.
3. China wants to avoid financial collapse of their economy. In order to do this, China must raise their gross national product rate by 33 percent. China's economy is structured so that if the 33 percent increase in GNP is reached, then it is possible for a 50 percent GNP increase.
Of the following statements listed below, which one must be true if is it to be believed that the above statements are also true?
A. If China's 50 percent increase in GNP is unattainable, then its economy will collapse.
B. China's GNP will not have a 50 percent increase if its economy falls.
C. The economy of China will not fall if it can obtain an increased GNP of 50 percent.
D. A 17 percent GNP increase will be unattainable if China continues to suffer national conflict.
E. A 71 percent increase is possible if the 33 percent brink is achieved, and the 50 percent GNP increase is attainable.
4. Estelle states: When I went fishing the other day, every fish that I caught was a salmon, and every salmon I saw I caught.
Of the following statements listed below, which one can be concluded from the observations of Estelle?
A. Salmon was the only fish that Estelle saw while she was fishing.
B. While Estelle was fishing, she caught no fish other than salmon.
C. In the area that Estelle fished, there were no other fish.
D. All of the fish that Estelle saw she caught.
E. Estelle did not see any other fish while she was fishing.
5. Either protesters must have restrictions placed on them, or particular revolutionary issues that arise in society will be used to destroy the country. Because allowing the occurrence of the revolutionary outcome is out of the question, we must restrict protestors.
The above conclusion is unsteady due to the fact that
A. Protestors do not really want to demolish the country.
B. There is too much emphasis placed on the importance of protesting.
C. An accommodation is failed to be considered between both alternatives.
D. The reasons for protesting have not been defined.
E. Protestors are, in truth, a real threat to the country's survival.
6. The quarterly food inspection performed by the local health team observes the customer reactions to fast food restaurants and family dining restaurants. However, during each inspection they discover that there are more reports of food poisoning found in the family dining restaurants than in the fast food restaurants.
Of the following statements, which one best clarifies the apparent paradox?
A. Customers are most likely to connect the illness they've experienced to their most recent meal if the illness has suddenly struck all of the people they ate with.
B. Customers complain less about the food poisoning they experience in fast food restaurants because they expect it.
C. More people choose fast food restaurants over family dining restaurants.
D. Food poisoning cases are not related to the time customers ate at the family dining restaurants, or to the number of people who all ate the same meal.
E. The family dining restaurants microwaves certain food items instead of cooking them on the stove.
7. We are well aware that there are warning signs concerning massive climate changes, and that these climate changes are reducing plant life. Many hopeful crop growers believe that there will not be an overall negative effect on the plant growth population due to the fact that rainfall should not be altered because of the climate changes. However, for the average plant, it is because of the climate change that agricultural technology has an overall yield in annual fluctuation.
On which of the following assumptions are these claims based?
A. There is not an accurate way to predict a climate change.
B. If patterns of rainfall began to shadow the climate changes, there would be supplementary damaging effects.
C. Improved yields grow highly unlikely if technology is significantly influential in spite of climate change.
D. Rainfall patterns are not as predictable as patterns of temperature.
E. Plant life is threatened more from cool temperatures than warm ones.
8. Different life forms such as animals and plants are known to have certain breeds that have extended life spans compared to that of humans. Due to this fact, scientists pass away before they are able to study the complete life cycle of these certain breeds. This being the case, a single breed may be inferred by observation over various life stages. Geology or astronomy can be applied to using the same method. Scientists can also use this method to learn about desert evolution and rock formations.
Concerning this passage, what assumption can be made evident?
A. The average subject uses the same ideals of observation.
B. Certain endangered breeds need to be studied more before they become extinct.
C. Various stage developments of different breeds are available to scientists as examples for both study and observation.
D. Through the use of today's study technique there are many breeds in society that cannot be properly studied in the environment.
E. There are different techniques that scientists can use that are available in modern society.
9. A rich businessman runs a prosperous company. He is disappointed in his two children, Violet and Hazen, because he believes that neither of them presents the potential of having the ability to take control of his company. He thinks that both of his children lack common sense.
This belief formulates from the opinion that
A. Violet and Hazen are ignorant to the experience of controlling the company.
B. Even a person who is not brilliant can control a company if she or he has been able to obtain an MBA.
C. In order to run a company, a person needs common sense.
D. If Hazen showed any sign of common sense, he would have the ability to aid Violet in controlling the company.
E. A committee with an average of three trained personnel could assist either Violet or Hazen in controlling the company.
10. The regulation that is proposed for dental insurance will contribute only small assistance to patients in the reduction of costs for regular dental care. Although the bill limits the amount that the dentist can charge for a regular visit, it does not limit the expense amount that they can charge if a patient is in need of a special procedure, and it doesn't place any limits on the amount of times that the dentist may see a patient for the same occurring problem. This being the case, instead of the patient being charged once, the dentist can bill the patient numerous times, and the total costs will not reduce.
The above argument is opposed to the new legislation based on
A. Identifying a loophole in the regulation proposal that allows dentists to charge patients the same amount of money on a continuing basis.
B. Implication that the regulation of dental care is impossible.
C. The suggestion that procedures which are specialized are frequently done when a simple, less-expensive procedure would work equally as well.
D. The suggestion that regular dental visits are much more expensive than specialized procedures.
E. The suggestion that patients cannot distinguish between what appropriate dental care expenses there are for any known dental problem.
11. The past three consecutive women's U.S. tennis champions have all changed to Wilson's new line of tennis rackets, exclusively made of oak wood for greater strength and durability. If this is the case, don't you think it's time to improve your tennis swing and trade your old racket in for a Wilson?
Which of the following claims is not made and cannot be used in conclusion to the above advertisement?
A. Previous U.S. tennis champions know a considerable amount about their equipment and the sport of tennis.
B. Rackets that are strengthened by oak wood are used exclusively in Wilson's new rackets.
C. Oak-wood-strengthened rackets help to make tennis rackets durable and stronger, allowing the player to make powerful swings.
D. With Wilson's rackets, you will improve your tennis playing.
E. The status achieved by the past three consecutive women's U.S. tennis championships was due to the assistance of Wilson's rackets.
12. My family doctor said that he would be performing a blood test on me when I visit him today. I know I will feel pain today.
The above argument depends on which one of these assumptions?
A. The use of a needle always causes pain in the patient.
B. The doctor will have a hard time finding the patient's vein.
C. In the past, this patient has experienced pain at the family doctor.
D. The needle will leave a bruise.
E. The doctor will have to try different needles to perform the test.
13. An English school teacher requested her students to try and write children's stories that are relevant to their everyday lives. The idea would be to give their stories a quality of "real life". One of the students decided to base her story upon one of the fictional characters from her favorite novel.
Of the following criteria listed below, which would make the most logical sense as criticism against the student's choice for her children's story?
A. The writing techniques that are successful for one writer are frequently unsuccessful for another.
B. A story based entirely on the writer's knowledge of characters from another novel is not likely to include the writer's personal feelings of real life.
C. The author of her favorite novel would not allow the student permission to use their character.
D. Children's story writing requires examination of the self in order to develop innovative and original ideas.
E. A writer should concentrate on developing themes of teaching and significance, instead of simply aiming for prominence.
14. A monopoly is distinguished through the decline or lack of competition. The MANG Company recognizes that its operations are within a competitive field.
Of the following conclusions, which one may be an implication of the above statement?
A. A one-seller market is the definition of a monopoly.
B. There is no family competition in the MANG Company.
C. The MANG Company's focus is non-monopolistic.
D. The MANG Company operates within a service industry.
E. The MANG Company is owned publicly.
15. If Local AA (Alcoholics Anonymous) rehabilitation facilities are forced upon us-and society is determined that they should be forced-then it should be society that pays for them.
Which of the following statements would weaken the argument above?
A. Many groups have been convinced by a government committee that the local rehabilitation facilities are unsuccessful.
B. The cost of AA facilities is too high to be funded locally.
C. Rehabilitation facilities are supported by many neighborhood groups.
D. The expense to maintain an AA rehabilitation facility is too costly.
E. Alcoholics may not want to accept treatment.
Refer to the following passage for questions 16 and 17:
While traveling to Japan, a low-ranking US ambassador asked a Japanese official why Japanese people were so inscrutable. The official looked calm and friendly, responding in a gentle voice that he much preferred to think upon his race as inscrutable than of his race as wanting in perspicacity such as in Americans.
16. Which of the following statements best describes the Japanese official's comment?
A. All people are inscrutable, not just the Japanese.
B. Most Americans don't understand Japanese culture.
C. What a person lacks in perception may be a result of the carelessness of the observer, instead of the obscurity within the object being observed.
D. The Japanese distrust American ambassadors.
E. If the East and West are ever to understand one another, there will need to be a much better cultural understanding.
17. Which of the following words best describes both the attitude and the response made by the Japanese official?
A. Fearful
B. Emotional
C. Angry
D. Indifferent
E. Compassionate
18. One day, a poet was requested to interpret an especially peculiar and obscure passage within one of his own poems. His response was "at the time that I was writing that particular verse, only God and myself knew its meaning. Now, it is only God who knows."
What does the poet mean by his answer?
A. God is much wiser than people are.
B. Most people can't understand poetry.
C. Poets don't often know where their creative inspiration comes from.
D. Great poems are inspired by a muse.
E. The poet has forgotten the meaning of his own verse.
19. All birds have beaks, and all sparrows are birds, so all sparrows must have beaks.
What is the basis of this author's argument?
A. Generalization
B. Syllogism
C. Special training
D. Induction
E. Ambiguity
20. Never again will you have to pay high prices for imported spring water. It is now bottled locally and inexpensively. You'll never taste the difference, however. And if you're likely to be embarrassed to serve domestic spring water, simply serve it in a leaded crystal decanter.
What is the assumption made by this ad?
A. It's not hard to tell domestic water from imported water based on its flavor.
B. The majority of spring water is bottled at its source.
C. Restrictions on importing and customs duties make the price of imported water higher.
D. Spring water tastes best when it's served from a decanter.
E. Some people purchase imported spring water instead of domestic as a status symbol.
21. Priest: Do you speak to the devil and follow his biddings?
Parishioner: Yes.
Priest: You must be lying. Nobody who is in league with the devil tells the truth.
Why can the priest's behavior be considered paradoxical?
A. He accused the parishioner of being in league with the devil, but he later changed his story.
B. He relied upon the answer of the parishioner in order to reject his response.
C. His behavior was entirely within accordance with religious law, but he was accusing the parishioner of violating that law.
D. While he is questioning the parishioner about possible association with the devil, he doesn't actually believe in such a thing.
E. He was the one who asked the question, but he refused to accept the answer.
Answers and Explanations
1. C: If e-book subscriptions remained stable in the five preceding years while textbook purchases declined, this would support the explanation for textbook price increases in response to lower textbook sales. A publication backlog (A) would not raise prices. Student numbers declining (B) does not justify raising textbook prices when e-book sales did not decline. Electronic publishers would not reduce textbook subscriptions (D), since they don't publish these. More available textbooks (E) should lower, not raise, prices.
2. D: The statement that dumping in lower-fish areas is less regulated and less expensive weakens the argument that dumping there implies the dumped pesticide is dangerous, by supplying two other motivations for dumping there. Previous underestimation of danger (A) does not weaken and could support the argument for implicitly admitting danger. The passage does not identify the area as rural (B). (C) contradicts itself, making no sense. The passage does not identify the area as urban (E).
3. A: The passage states China must raise its GNP by 33% to avoid economic collapse; if 33% is reached, 50% is possible. If 50% is impossible, 33% was not reached; the economy will collapse. China's economy will fall without GNP increase, not vice versa (B); and by 33%, not 50% (B), (C). National conflict is not mentioned; neither is 17% (D) or 71% GNP increase (E).
4. B: If every fish Estelle caught was a salmon, all she caught were salmon. Salmon were not necessarily the only fish she saw (A); she could have seen but not caught other fish. Likewise there were not necessarily no other fish there (C); she just didn't catch them. Estelle caught all the salmon she saw, not all the fish she saw (D). (E) is the same as (A)
5. C: The range of compromises between destroying the country and restricting protestors is ignored. Some protestors do not want to destroy the country (A), but some may. Therefore, emphasis on protesting's importance does not necessarily weaken the conclusion (B). Defining reasons for protesting (D) is not required to support the conclusion, which is based on protestors destroying the country regardless of the issues. That protestors are truly threats (E) strengthens, not weakens, the conclusion.
6. A: More diners eating together are more likely to notice all subsequently experiencing food poisoning. Customers expecting food poisoning would not eat at fast food restaurants (B). Even if expecting more chance of it, they would still report occurrences. More people choosing fast food restaurants (C) would increase the probability of food poisoning complaints from fast food, not family dining, restaurants. Choice (D) simply contradicts (A). Microwaving (E) is not associated with food poisoning.
7. B: Although hopeful growers believe climate changes "should not" change rainfall, these cause annual agricultural yield fluctuations. This claim proceeds from the assumption that since yields already fluctuate from climate change, additional rainfall change would exacerbate fluctuations. That climate change cannot be accurately predicted (A) is not claimed. Technology's significant influence despite climate change makes improved yields likely, NOT "highly unlikely" (C). That rainfall is less predictable than temperature (D), or cool temperatures more threatening (E), are not claims.
8. C: Scientists can observe and study various life stages of breeds too long-lived to study their full life span during scientists' lifetimes. The passage indicates subjects' using the same observational methods, but using the same ideals (A) cannot be assumed. Choice (B) is true, but not indicated in the passage. Today's techniques preventing studying many breeds in the environment (D) is nowhere suggested. Various scientific techniques are available (E), but this cannot be inferred from the passage.
9. C: He believes neither child can take control of his company because of his opinion that they "lack common sense." Therefore, he believes a person needs common sense to run a company. That his children are "ignorant to the experience of controlling the company" (A) is not indicated as his opinion. Nothing is mentioned about obtaining an MBA (B). He thinks both children lack common sense, not one (D). Committee assistance (E) is never mentioned.
10. A: The argument identifies the loophole of unlimited special procedure expenses, and unlimited repeat visit charges for the same problem. It does not imply dental care cannot be regulated (B). Substituting specialized procedures for less-expensive procedures (C) is never suggested. The passage never suggests that regular visits cost more than specialized procedures (D), or that patients cannot distinguish appropriate expenses (E).
11. E: Champions "have all changed to" Wilson's new rackets; they did not win past championships with them. Champions are knowledgeable about tennis and their equipment (A): the ad uses their choosing Wilson's as an example to follow. It indicates exclusive use of oak in Wilson's new line (B). Durability and strength are named as oak's benefits; from the suggestion these will "improve your tennis swing," we can infer "powerful swings" (C) and "you will improve your tennis playing" (D).
12. A: To know one will feel pain from a blood test, we must assume needles always cause this patient pain. Having trouble finding the vein (B) is not the only cause of needle pain. Experiencing pain with this doctor before (C) does not guarantee s/he always will. Leaving a bruise (D) does not always mean the needle hurt. Having to try different needles (E) is not the only cause of pain.
13. B: The student's real-life experience is unlikely to show in writing about fictional characters. Different writing techniques' variable success among writers (A) is not criticism of her choice. Author permission (C) is irrelevant to her choice's inappropriateness. Self-examination is important to originality (D), but misses the point: relevance to students' everyday lives, not originality, was assigned. Instructive, significant themes' precedence over fame (E) is irrelevant to her choosing fictional characters to describe her real-life experiences.
14. C: Monopoly means declining/ therefore, lacking competition. By recognizing its field is competitive, the company accepts competition and has a non-monopolistic focus. Monopoly's definition as a one-seller market (A) is not implied: the statement indicates only that monopoly is "distinguished through the decline or lack of competition." Family competition (B), service industry (D), and public ownership (E) are never implied in the statement.
15. A: If society believes local AA facilities are unsuccessful, society will not fund them. Cost exceeding local funding (B) does not weaken the argument: "society" does not necessarily mean local (and more likely means federal funding.) Neighborhood group support (C) would strengthen, not weaken, the argument for local facilities. Maintenance cost (D) strengthens the argument that if society wants to force facilities on communities, society should pay for them. Alcoholics' treatment resistance (E) is irrelevant to forcing/funding facilities.
16. C: The official attributed Americans' perception of Japanese as "inscrutable" to American lack of discernment, not Japanese inaccessibility. He never suggested all people are inscrutable (A). He did not say that most Americans don't understand Japanese culture specifically (B), but that Americans lack understanding generally. He never said the Japanese mistrust American ambassadors (D). For the offensive "inscrutable" stereotype, he returned a reciprocal opinion of Americans as unperceptive, rather than calling for improved cultural understanding (E).
17. E: The official's response and attitude were compassionate, indicated by his "calm and friendly" look and "gentle voice." His words also demonstrated a sense of humor. The description of his behavior is not fearful (A), emotional (B), angry (C), or indifferent (D). While he deflected a racial stereotype of the Japanese by introducing a corresponding one of Americans, he is not described as doing so with fear, emotion, anger, or indifference.
18. E: The poet humorously couches the fact that he forgot what he meant by saying only he and God knew at the time, and now only God knows-i.e., now he himself does not know. He is not saying God is wiser (A), but remembers better. He never says people can't understand poetry (B); he refers only to himself not knowing/remembering his meaning. He refers only to the verse's meaning, not its inspiration (C), (D).
19. B: This is a syllogism. Syllogisms have three parts: major premise, minor premise, conclusion. Syllogism uses deductive reasoning, reducing general information (all birds have beaks, all sparrows are birds) to infer a more specific conclusion (all sparrows have beaks). Inductive reasoning (D), the opposite of deductive, accumulates specific facts to form generalizations (A). No special training (C) was required to make this conclusion. The argument involves no ambiguity (E): it is not open to multiple interpretations or unclear.
20. E: The ad assumes some people find imported spring water a status symbol by mentioning their embarrassment over serving domestic. It assumes it is hard to tell these apart by flavor (A): "You'll never taste the difference". There is no assumption most spring water is bottled at its source (B). The only reason the ad assumes for higher prices is status, not regulations (C). The decanter conceals water's domestic origin rather than improving its taste (D).
21. B: The paradox is the priest using the parishioner's answer as a criterion for judging itself! He did not later change his accusation (A). Following/violating religious law (C) is irrelevant. There is no reason to think the priest doesn't believe in the devil (D). The questioner rejecting the answer (E) is not necessarily paradoxical if he doesn't like the answer. (This reflects the familiar conundrum: "Are you lying?" "Yes." "Then you must be telling the truth").
1. There has been a sharp increase in the subscription prices of many accounting school text books in the past five years. Many publishers ascribe the necessity for these increases to the easy availability of electronic books, which enable people to electronically copy the books they want rather than buying the printed text.
B. Over the past five years there has been a substantial decline in the number of accounting school students, while electronic books have remained fairly stable.
C. In the five years immediately preceding the price surge, there was a substantial decline in the number of accounting school students requiring text books, while electronic book subscriptions remained fairly stable.
D. Many electronic publishers have recently begun cutting back on subscriptions of accounting school text books.
E. In almost every publishing company, there has been an increase in the number of accounting school texts available in the past few years.
2. A pesticide producing company states that their unused pesticide that gets dumped does not pose a threat to the aquatic life in the surrounding area. If this is correct, then why have local fish been dying in this region? Due to the fact that the pesticide company is not located in a highly fish-populated area, they implicitly admit that the pesticides they produce are relatively dangerous to the nearby aquatic life.
Of the following statements listed below, which one would be most likely to weaken the argument of the author if it were true?
B. Funds for environmental company cleanup, which concern waste dumps that are poorly run, are reserved for rural regions only.
C. It would be pointless to locate chemical dumps where they would be most harmful, unless they can be proven 100-percent safe.
D. Dumps that are located in areas without large fish populations have fewer government interventions and are also less expensive.
3. China wants to avoid financial collapse of their economy. In order to do this, China must raise their gross national product rate by 33 percent. China's economy is structured so that if the 33 percent increase in GNP is reached, then it is possible for a 50 percent GNP increase.
Of the following statements listed below, which one must be true if is it to be believed that the above statements are also true?
E. A 71 percent increase is possible if the 33 percent brink is achieved, and the 50 percent GNP increase is attainable.
4. Estelle states: When I went fishing the other day, every fish that I caught was a salmon, and every salmon I saw I caught.
Of the following statements listed below, which one can be concluded from the observations of Estelle?
5. Either protesters must have restrictions placed on them, or particular revolutionary issues that arise in society will be used to destroy the country. Because allowing the occurrence of the revolutionary outcome is out of the question, we must restrict protestors.
6. The quarterly food inspection performed by the local health team observes the customer reactions to fast food restaurants and family dining restaurants. However, during each inspection they discover that there are more reports of food poisoning found in the family dining restaurants than in the fast food restaurants.
A. Customers are most likely to connect the illness they've experienced to their most recent meal if the illness has suddenly struck all of the people they ate with.
B. Customers complain less about the food poisoning they experience in fast food restaurants because they expect it.
D. Food poisoning cases are not related to the time customers ate at the family dining restaurants, or to the number of people who all ate the same meal.
7. We are well aware that there are warning signs concerning massive climate changes, and that these climate changes are reducing plant life. Many hopeful crop growers believe that there will not be an overall negative effect on the plant growth population due to the fact that rainfall should not be altered because of the climate changes. However, for the average plant, it is because of the climate change that agricultural technology has an overall yield in annual fluctuation.
B. If patterns of rainfall began to shadow the climate changes, there would be supplementary damaging effects.
C. Improved yields grow highly unlikely if technology is significantly influential in spite of climate change.
8. Different life forms such as animals and plants are known to have certain breeds that have extended life spans compared to that of humans. Due to this fact, scientists pass away before they are able to study the complete life cycle of these certain breeds. This being the case, a single breed may be inferred by observation over various life stages. Geology or astronomy can be applied to using the same method. Scientists can also use this method to learn about desert evolution and rock formations.
C. Various stage developments of different breeds are available to scientists as examples for both study and observation.
D. Through the use of today's study technique there are many breeds in society that cannot be properly studied in the environment.
9. A rich businessman runs a prosperous company. He is disappointed in his two children, Violet and Hazen, because he believes that neither of them presents the potential of having the ability to take control of his company. He thinks that both of his children lack common sense.
B. Even a person who is not brilliant can control a company if she or he has been able to obtain an MBA.
D. If Hazen showed any sign of common sense, he would have the ability to aid Violet in controlling the company.
E. A committee with an average of three trained personnel could assist either Violet or Hazen in controlling the company.
10. The regulation that is proposed for dental insurance will contribute only small assistance to patients in the reduction of costs for regular dental care. Although the bill limits the amount that the dentist can charge for a regular visit, it does not limit the expense amount that they can charge if a patient is in need of a special procedure, and it doesn't place any limits on the amount of times that the dentist may see a patient for the same occurring problem. This being the case, instead of the patient being charged once, the dentist can bill the patient numerous times, and the total costs will not reduce.
A. Identifying a loophole in the regulation proposal that allows dentists to charge patients the same amount of money on a continuing basis.
C. The suggestion that procedures which are specialized are frequently done when a simple, less-expensive procedure would work equally as well.
E. The suggestion that patients cannot distinguish between what appropriate dental care expenses there are for any known dental problem.
11. The past three consecutive women's U.S. tennis champions have all changed to Wilson's new line of tennis rackets, exclusively made of oak wood for greater strength and durability. If this is the case, don't you think it's time to improve your tennis swing and trade your old racket in for a Wilson?
Which of the following claims is not made and cannot be used in conclusion to the above advertisement?
A. Previous U.S. tennis champions know a considerable amount about their equipment and the sport of tennis.
C. Oak-wood-strengthened rackets help to make tennis rackets durable and stronger, allowing the player to make powerful swings.
E. The status achieved by the past three consecutive women's U.S. tennis championships was due to the assistance of Wilson's rackets.
12. My family doctor said that he would be performing a blood test on me when I visit him today. I know I will feel pain today.
13. An English school teacher requested her students to try and write children's stories that are relevant to their everyday lives. The idea would be to give their stories a quality of "real life". One of the students decided to base her story upon one of the fictional characters from her favorite novel.
Of the following criteria listed below, which would make the most logical sense as criticism against the student's choice for her children's story?
B. A story based entirely on the writer's knowledge of characters from another novel is not likely to include the writer's personal feelings of real life.
D. Children's story writing requires examination of the self in order to develop innovative and original ideas.
E. A writer should concentrate on developing themes of teaching and significance, instead of simply aiming for prominence.
14. A monopoly is distinguished through the decline or lack of competition. The MANG Company recognizes that its operations are within a competitive field.
15. If Local AA (Alcoholics Anonymous) rehabilitation facilities are forced upon us-and society is determined that they should be forced-then it should be society that pays for them.
A. Many groups have been convinced by a government committee that the local rehabilitation facilities are unsuccessful.
While traveling to Japan, a low-ranking US ambassador asked a Japanese official why Japanese people were so inscrutable. The official looked calm and friendly, responding in a gentle voice that he much preferred to think upon his race as inscrutable than of his race as wanting in perspicacity such as in Americans.
C. What a person lacks in perception may be a result of the carelessness of the observer, instead of the obscurity within the object being observed.
E. If the East and West are ever to understand one another, there will need to be a much better cultural understanding.
17. Which of the following words best describes both the attitude and the response made by the Japanese official?
18. One day, a poet was requested to interpret an especially peculiar and obscure passage within one of his own poems. His response was "at the time that I was writing that particular verse, only God and myself knew its meaning. Now, it is only God who knows."
20. Never again will you have to pay high prices for imported spring water. It is now bottled locally and inexpensively. You'll never taste the difference, however. And if you're likely to be embarrassed to serve domestic spring water, simply serve it in a leaded crystal decanter.
C. His behavior was entirely within accordance with religious law, but he was accusing the parishioner of violating that law.
D. While he is questioning the parishioner about possible association with the devil, he doesn't actually believe in such a thing.
1. C: If e-book subscriptions remained stable in the five preceding years while textbook purchases declined, this would support the explanation for textbook price increases in response to lower textbook sales. A publication backlog (A) would not raise prices. Student numbers declining (B) does not justify raising textbook prices when e-book sales did not decline. Electronic publishers would not reduce textbook subscriptions (D), since they don't publish these. More available textbooks (E) should lower, not raise, prices.
2. D: The statement that dumping in lower-fish areas is less regulated and less expensive weakens the argument that dumping there implies the dumped pesticide is dangerous, by supplying two other motivations for dumping there. Previous underestimation of danger (A) does not weaken and could support the argument for implicitly admitting danger. The passage does not identify the area as rural (B). (C) contradicts itself, making no sense. The passage does not identify the area as urban (E).
3. A: The passage states China must raise its GNP by 33% to avoid economic collapse; if 33% is reached, 50% is possible. If 50% is impossible, 33% was not reached; the economy will collapse. China's economy will fall without GNP increase, not vice versa (B); and by 33%, not 50% (B), (C). National conflict is not mentioned; neither is 17% (D) or 71% GNP increase (E).
4. B: If every fish Estelle caught was a salmon, all she caught were salmon. Salmon were not necessarily the only fish she saw (A); she could have seen but not caught other fish. Likewise there were not necessarily no other fish there (C); she just didn't catch them. Estelle caught all the salmon she saw, not all the fish she saw (D). (E) is the same as (A)
5. C: The range of compromises between destroying the country and restricting protestors is ignored. Some protestors do not want to destroy the country (A), but some may. Therefore, emphasis on protesting's importance does not necessarily weaken the conclusion (B). Defining reasons for protesting (D) is not required to support the conclusion, which is based on protestors destroying the country regardless of the issues. That protestors are truly threats (E) strengthens, not weakens, the conclusion.
6. A: More diners eating together are more likely to notice all subsequently experiencing food poisoning. Customers expecting food poisoning would not eat at fast food restaurants (B). Even if expecting more chance of it, they would still report occurrences. More people choosing fast food restaurants (C) would increase the probability of food poisoning complaints from fast food, not family dining, restaurants. Choice (D) simply contradicts (A). Microwaving (E) is not associated with food poisoning.
7. B: Although hopeful growers believe climate changes "should not" change rainfall, these cause annual agricultural yield fluctuations. This claim proceeds from the assumption that since yields already fluctuate from climate change, additional rainfall change would exacerbate fluctuations. That climate change cannot be accurately predicted (A) is not claimed. Technology's significant influence despite climate change makes improved yields likely, NOT "highly unlikely" (C). That rainfall is less predictable than temperature (D), or cool temperatures more threatening (E), are not claims.
8. C: Scientists can observe and study various life stages of breeds too long-lived to study their full life span during scientists' lifetimes. The passage indicates subjects' using the same observational methods, but using the same ideals (A) cannot be assumed. Choice (B) is true, but not indicated in the passage. Today's techniques preventing studying many breeds in the environment (D) is nowhere suggested. Various scientific techniques are available (E), but this cannot be inferred from the passage.
9. C: He believes neither child can take control of his company because of his opinion that they "lack common sense." Therefore, he believes a person needs common sense to run a company. That his children are "ignorant to the experience of controlling the company" (A) is not indicated as his opinion. Nothing is mentioned about obtaining an MBA (B). He thinks both children lack common sense, not one (D). Committee assistance (E) is never mentioned.
10. A: The argument identifies the loophole of unlimited special procedure expenses, and unlimited repeat visit charges for the same problem. It does not imply dental care cannot be regulated (B). Substituting specialized procedures for less-expensive procedures (C) is never suggested. The passage never suggests that regular visits cost more than specialized procedures (D), or that patients cannot distinguish appropriate expenses (E).
11. E: Champions "have all changed to" Wilson's new rackets; they did not win past championships with them. Champions are knowledgeable about tennis and their equipment (A): the ad uses their choosing Wilson's as an example to follow. It indicates exclusive use of oak in Wilson's new line (B). Durability and strength are named as oak's benefits; from the suggestion these will "improve your tennis swing," we can infer "powerful swings" (C) and "you will improve your tennis playing" (D).
12. A: To know one will feel pain from a blood test, we must assume needles always cause this patient pain. Having trouble finding the vein (B) is not the only cause of needle pain. Experiencing pain with this doctor before (C) does not guarantee s/he always will. Leaving a bruise (D) does not always mean the needle hurt. Having to try different needles (E) is not the only cause of pain.
13. B: The student's real-life experience is unlikely to show in writing about fictional characters. Different writing techniques' variable success among writers (A) is not criticism of her choice. Author permission (C) is irrelevant to her choice's inappropriateness. Self-examination is important to originality (D), but misses the point: relevance to students' everyday lives, not originality, was assigned. Instructive, significant themes' precedence over fame (E) is irrelevant to her choosing fictional characters to describe her real-life experiences.
14. C: Monopoly means declining/ therefore, lacking competition. By recognizing its field is competitive, the company accepts competition and has a non-monopolistic focus. Monopoly's definition as a one-seller market (A) is not implied: the statement indicates only that monopoly is "distinguished through the decline or lack of competition." Family competition (B), service industry (D), and public ownership (E) are never implied in the statement.
15. A: If society believes local AA facilities are unsuccessful, society will not fund them. Cost exceeding local funding (B) does not weaken the argument: "society" does not necessarily mean local (and more likely means federal funding.) Neighborhood group support (C) would strengthen, not weaken, the argument for local facilities. Maintenance cost (D) strengthens the argument that if society wants to force facilities on communities, society should pay for them. Alcoholics' treatment resistance (E) is irrelevant to forcing/funding facilities.
16. C: The official attributed Americans' perception of Japanese as "inscrutable" to American lack of discernment, not Japanese inaccessibility. He never suggested all people are inscrutable (A). He did not say that most Americans don't understand Japanese culture specifically (B), but that Americans lack understanding generally. He never said the Japanese mistrust American ambassadors (D). For the offensive "inscrutable" stereotype, he returned a reciprocal opinion of Americans as unperceptive, rather than calling for improved cultural understanding (E).
17. E: The official's response and attitude were compassionate, indicated by his "calm and friendly" look and "gentle voice." His words also demonstrated a sense of humor. The description of his behavior is not fearful (A), emotional (B), angry (C), or indifferent (D). While he deflected a racial stereotype of the Japanese by introducing a corresponding one of Americans, he is not described as doing so with fear, emotion, anger, or indifference.
18. E: The poet humorously couches the fact that he forgot what he meant by saying only he and God knew at the time, and now only God knows-i.e., now he himself does not know. He is not saying God is wiser (A), but remembers better. He never says people can't understand poetry (B); he refers only to himself not knowing/remembering his meaning. He refers only to the verse's meaning, not its inspiration (C), (D).
19. B: This is a syllogism. Syllogisms have three parts: major premise, minor premise, conclusion. Syllogism uses deductive reasoning, reducing general information (all birds have beaks, all sparrows are birds) to infer a more specific conclusion (all sparrows have beaks). Inductive reasoning (D), the opposite of deductive, accumulates specific facts to form generalizations (A). No special training (C) was required to make this conclusion. The argument involves no ambiguity (E): it is not open to multiple interpretations or unclear.
20. E: The ad assumes some people find imported spring water a status symbol by mentioning their embarrassment over serving domestic. It assumes it is hard to tell these apart by flavor (A): "You'll never taste the difference". There is no assumption most spring water is bottled at its source (B). The only reason the ad assumes for higher prices is status, not regulations (C). The decanter conceals water's domestic origin rather than improving its taste (D).
21. B: The paradox is the priest using the parishioner's answer as a criterion for judging itself! He did not later change his accusation (A). Following/violating religious law (C) is irrelevant. There is no reason to think the priest doesn't believe in the devil (D). The questioner rejecting the answer (E) is not necessarily paradoxical if he doesn't like the answer. (This reflects the familiar conundrum: "Are you lying?" "Yes." "Then you must be telling the truth").
1. There has been a sharp increase in the subscription prices of many accounting school text books in the past five years. Many publishers ascribe the necessity for these increases to the easy availability of electronic books, which enable people to electronically copy the books they want rather than buying the printed text.
C. In the five years immediately preceding the price surge, there was a substantial decline in the number of accounting school students requiring text books, while electronic book subscriptions remained fairly stable.
2. A pesticide producing company states that their unused pesticide that gets dumped does not pose a threat to the aquatic life in the surrounding area. If this is correct, then why have local fish been dying in this region? Due to the fact that the pesticide company is not located in a highly fish-populated area, they implicitly admit that the pesticides they produce are relatively dangerous to the nearby aquatic life.
3. China wants to avoid financial collapse of their economy. In order to do this, China must raise their gross national product rate by 33 percent. China's economy is structured so that if the 33 percent increase in GNP is reached, then it is possible for a 50 percent GNP increase.
5. Either protesters must have restrictions placed on them, or particular revolutionary issues that arise in society will be used to destroy the country. Because allowing the occurrence of the revolutionary outcome is out of the question, we must restrict protestors.
6. The quarterly food inspection performed by the local health team observes the customer reactions to fast food restaurants and family dining restaurants. However, during each inspection they discover that there are more reports of food poisoning found in the family dining restaurants than in the fast food restaurants.
7. We are well aware that there are warning signs concerning massive climate changes, and that these climate changes are reducing plant life. Many hopeful crop growers believe that there will not be an overall negative effect on the plant growth population due to the fact that rainfall should not be altered because of the climate changes. However, for the average plant, it is because of the climate change that agricultural technology has an overall yield in annual fluctuation.
8. Different life forms such as animals and plants are known to have certain breeds that have extended life spans compared to that of humans. Due to this fact, scientists pass away before they are able to study the complete life cycle of these certain breeds. This being the case, a single breed may be inferred by observation over various life stages. Geology or astronomy can be applied to using the same method. Scientists can also use this method to learn about desert evolution and rock formations.
9. A rich businessman runs a prosperous company. He is disappointed in his two children, Violet and Hazen, because he believes that neither of them presents the potential of having the ability to take control of his company. He thinks that both of his children lack common sense.
10. The regulation that is proposed for dental insurance will contribute only small assistance to patients in the reduction of costs for regular dental care. Although the bill limits the amount that the dentist can charge for a regular visit, it does not limit the expense amount that they can charge if a patient is in need of a special procedure, and it doesn't place any limits on the amount of times that the dentist may see a patient for the same occurring problem. This being the case, instead of the patient being charged once, the dentist can bill the patient numerous times, and the total costs will not reduce.
11. The past three consecutive women's U.S. tennis champions have all changed to Wilson's new line of tennis rackets, exclusively made of oak wood for greater strength and durability. If this is the case, don't you think it's time to improve your tennis swing and trade your old racket in for a Wilson?
13. An English school teacher requested her students to try and write children's stories that are relevant to their everyday lives. The idea would be to give their stories a quality of "real life". One of the students decided to base her story upon one of the fictional characters from her favorite novel.
While traveling to Japan, a low-ranking US ambassador asked a Japanese official why Japanese people were so inscrutable. The official looked calm and friendly, responding in a gentle voice that he much preferred to think upon his race as inscrutable than of his race as wanting in perspicacity such as in Americans.
18. One day, a poet was requested to interpret an especially peculiar and obscure passage within one of his own poems. His response was "at the time that I was writing that particular verse, only God and myself knew its meaning. Now, it is only God who knows."
20. Never again will you have to pay high prices for imported spring water. It is now bottled locally and inexpensively. You'll never taste the difference, however. And if you're likely to be embarrassed to serve domestic spring water, simply serve it in a leaded crystal decanter.
1. C: If e-book subscriptions remained stable in the five preceding years while textbook purchases declined, this would support the explanation for textbook price increases in response to lower textbook sales. A publication backlog (A) would not raise prices. Student numbers declining (B) does not justify raising textbook prices when e-book sales did not decline. Electronic publishers would not reduce textbook subscriptions (D), since they don't publish these. More available textbooks (E) should lower, not raise, prices.
2. D: The statement that dumping in lower-fish areas is less regulated and less expensive weakens the argument that dumping there implies the dumped pesticide is dangerous, by supplying two other motivations for dumping there. Previous underestimation of danger (A) does not weaken and could support the argument for implicitly admitting danger. The passage does not identify the area as rural (B). (C) contradicts itself, making no sense. The passage does not identify the area as urban (E).
3. A: The passage states China must raise its GNP by 33% to avoid economic collapse; if 33% is reached, 50% is possible. If 50% is impossible, 33% was not reached; the economy will collapse. China's economy will fall without GNP increase, not vice versa (B); and by 33%, not 50% (B), (C). National conflict is not mentioned; neither is 17% (D) or 71% GNP increase (E).
4. B: If every fish Estelle caught was a salmon, all she caught were salmon. Salmon were not necessarily the only fish she saw (A); she could have seen but not caught other fish. Likewise there were not necessarily no other fish there (C); she just didn't catch them. Estelle caught all the salmon she saw, not all the fish she saw (D). (E) is the same as (A)
5. C: The range of compromises between destroying the country and restricting protestors is ignored. Some protestors do not want to destroy the country (A), but some may. Therefore, emphasis on protesting's importance does not necessarily weaken the conclusion (B). Defining reasons for protesting (D) is not required to support the conclusion, which is based on protestors destroying the country regardless of the issues. That protestors are truly threats (E) strengthens, not weakens, the conclusion.
6. A: More diners eating together are more likely to notice all subsequently experiencing food poisoning. Customers expecting food poisoning would not eat at fast food restaurants (B). Even if expecting more chance of it, they would still report occurrences. More people choosing fast food restaurants (C) would increase the probability of food poisoning complaints from fast food, not family dining, restaurants. Choice (D) simply contradicts (A). Microwaving (E) is not associated with food poisoning.
7. B: Although hopeful growers believe climate changes "should not" change rainfall, these cause annual agricultural yield fluctuations. This claim proceeds from the assumption that since yields already fluctuate from climate change, additional rainfall change would exacerbate fluctuations. That climate change cannot be accurately predicted (A) is not claimed. Technology's significant influence despite climate change makes improved yields likely, NOT "highly unlikely" (C). That rainfall is less predictable than temperature (D), or cool temperatures more threatening (E), are not claims.
8. C: Scientists can observe and study various life stages of breeds too long-lived to study their full life span during scientists' lifetimes. The passage indicates subjects' using the same observational methods, but using the same ideals (A) cannot be assumed. Choice (B) is true, but not indicated in the passage. Today's techniques preventing studying many breeds in the environment (D) is nowhere suggested. Various scientific techniques are available (E), but this cannot be inferred from the passage.
9. C: He believes neither child can take control of his company because of his opinion that they "lack common sense." Therefore, he believes a person needs common sense to run a company. That his children are "ignorant to the experience of controlling the company" (A) is not indicated as his opinion. Nothing is mentioned about obtaining an MBA (B). He thinks both children lack common sense, not one (D). Committee assistance (E) is never mentioned.
10. A: The argument identifies the loophole of unlimited special procedure expenses, and unlimited repeat visit charges for the same problem. It does not imply dental care cannot be regulated (B). Substituting specialized procedures for less-expensive procedures (C) is never suggested. The passage never suggests that regular visits cost more than specialized procedures (D), or that patients cannot distinguish appropriate expenses (E).
11. E: Champions "have all changed to" Wilson's new rackets; they did not win past championships with them. Champions are knowledgeable about tennis and their equipment (A): the ad uses their choosing Wilson's as an example to follow. It indicates exclusive use of oak in Wilson's new line (B). Durability and strength are named as oak's benefits; from the suggestion these will "improve your tennis swing," we can infer "powerful swings" (C) and "you will improve your tennis playing" (D).
12. A: To know one will feel pain from a blood test, we must assume needles always cause this patient pain. Having trouble finding the vein (B) is not the only cause of needle pain. Experiencing pain with this doctor before (C) does not guarantee s/he always will. Leaving a bruise (D) does not always mean the needle hurt. Having to try different needles (E) is not the only cause of pain.
13. B: The student's real-life experience is unlikely to show in writing about fictional characters. Different writing techniques' variable success among writers (A) is not criticism of her choice. Author permission (C) is irrelevant to her choice's inappropriateness. Self-examination is important to originality (D), but misses the point: relevance to students' everyday lives, not originality, was assigned. Instructive, significant themes' precedence over fame (E) is irrelevant to her choosing fictional characters to describe her real-life experiences.
14. C: Monopoly means declining/ therefore, lacking competition. By recognizing its field is competitive, the company accepts competition and has a non-monopolistic focus. Monopoly's definition as a one-seller market (A) is not implied: the statement indicates only that monopoly is "distinguished through the decline or lack of competition." Family competition (B), service industry (D), and public ownership (E) are never implied in the statement.
15. A: If society believes local AA facilities are unsuccessful, society will not fund them. Cost exceeding local funding (B) does not weaken the argument: "society" does not necessarily mean local (and more likely means federal funding.) Neighborhood group support (C) would strengthen, not weaken, the argument for local facilities. Maintenance cost (D) strengthens the argument that if society wants to force facilities on communities, society should pay for them. Alcoholics' treatment resistance (E) is irrelevant to forcing/funding facilities.
16. C: The official attributed Americans' perception of Japanese as "inscrutable" to American lack of discernment, not Japanese inaccessibility. He never suggested all people are inscrutable (A). He did not say that most Americans don't understand Japanese culture specifically (B), but that Americans lack understanding generally. He never said the Japanese mistrust American ambassadors (D). For the offensive "inscrutable" stereotype, he returned a reciprocal opinion of Americans as unperceptive, rather than calling for improved cultural understanding (E).
17. E: The official's response and attitude were compassionate, indicated by his "calm and friendly" look and "gentle voice." His words also demonstrated a sense of humor. The description of his behavior is not fearful (A), emotional (B), angry (C), or indifferent (D). While he deflected a racial stereotype of the Japanese by introducing a corresponding one of Americans, he is not described as doing so with fear, emotion, anger, or indifference.
18. E: The poet humorously couches the fact that he forgot what he meant by saying only he and God knew at the time, and now only God knows-i.e., now he himself does not know. He is not saying God is wiser (A), but remembers better. He never says people can't understand poetry (B); he refers only to himself not knowing/remembering his meaning. He refers only to the verse's meaning, not its inspiration (C), (D).
19. B: This is a syllogism. Syllogisms have three parts: major premise, minor premise, conclusion. Syllogism uses deductive reasoning, reducing general information (all birds have beaks, all sparrows are birds) to infer a more specific conclusion (all sparrows have beaks). Inductive reasoning (D), the opposite of deductive, accumulates specific facts to form generalizations (A). No special training (C) was required to make this conclusion. The argument involves no ambiguity (E): it is not open to multiple interpretations or unclear.
20. E: The ad assumes some people find imported spring water a status symbol by mentioning their embarrassment over serving domestic. It assumes it is hard to tell these apart by flavor (A): "You'll never taste the difference". There is no assumption most spring water is bottled at its source (B). The only reason the ad assumes for higher prices is status, not regulations (C). The decanter conceals water's domestic origin rather than improving its taste (D).
21. B: The paradox is the priest using the parishioner's answer as a criterion for judging itself! He did not later change his accusation (A). Following/violating religious law (C) is irrelevant. There is no reason to think the priest doesn't believe in the devil (D). The questioner rejecting the answer (E) is not necessarily paradoxical if he doesn't like the answer. (This reflects the familiar conundrum: "Are you lying?" "Yes." "Then you must be telling the truth").
1. There has been a sharp increase in the subscription prices of many accounting school text books in the past five years. Many publishers ascribe the necessity for these increases to the easy availability of electronic books, which enable people to electronically copy the books they want rather than buying the printed text.
2. A pesticide producing company states that their unused pesticide that gets dumped does not pose a threat to the aquatic life in the surrounding area. If this is correct, then why have local fish been dying in this region? Due to the fact that the pesticide company is not located in a highly fish-populated area, they implicitly admit that the pesticides they produce are relatively dangerous to the nearby aquatic life.
6. The quarterly food inspection performed by the local health team observes the customer reactions to fast food restaurants and family dining restaurants. However, during each inspection they discover that there are more reports of food poisoning found in the family dining restaurants than in the fast food restaurants.
7. We are well aware that there are warning signs concerning massive climate changes, and that these climate changes are reducing plant life. Many hopeful crop growers believe that there will not be an overall negative effect on the plant growth population due to the fact that rainfall should not be altered because of the climate changes. However, for the average plant, it is because of the climate change that agricultural technology has an overall yield in annual fluctuation.
8. Different life forms such as animals and plants are known to have certain breeds that have extended life spans compared to that of humans. Due to this fact, scientists pass away before they are able to study the complete life cycle of these certain breeds. This being the case, a single breed may be inferred by observation over various life stages. Geology or astronomy can be applied to using the same method. Scientists can also use this method to learn about desert evolution and rock formations.
10. The regulation that is proposed for dental insurance will contribute only small assistance to patients in the reduction of costs for regular dental care. Although the bill limits the amount that the dentist can charge for a regular visit, it does not limit the expense amount that they can charge if a patient is in need of a special procedure, and it doesn't place any limits on the amount of times that the dentist may see a patient for the same occurring problem. This being the case, instead of the patient being charged once, the dentist can bill the patient numerous times, and the total costs will not reduce.
11. The past three consecutive women's U.S. tennis champions have all changed to Wilson's new line of tennis rackets, exclusively made of oak wood for greater strength and durability. If this is the case, don't you think it's time to improve your tennis swing and trade your old racket in for a Wilson?
13. An English school teacher requested her students to try and write children's stories that are relevant to their everyday lives. The idea would be to give their stories a quality of "real life". One of the students decided to base her story upon one of the fictional characters from her favorite novel.
While traveling to Japan, a low-ranking US ambassador asked a Japanese official why Japanese people were so inscrutable. The official looked calm and friendly, responding in a gentle voice that he much preferred to think upon his race as inscrutable than of his race as wanting in perspicacity such as in Americans.
1. C: If e-book subscriptions remained stable in the five preceding years while textbook purchases declined, this would support the explanation for textbook price increases in response to lower textbook sales. A publication backlog (A) would not raise prices. Student numbers declining (B) does not justify raising textbook prices when e-book sales did not decline. Electronic publishers would not reduce textbook subscriptions (D), since they don't publish these. More available textbooks (E) should lower, not raise, prices.
2. D: The statement that dumping in lower-fish areas is less regulated and less expensive weakens the argument that dumping there implies the dumped pesticide is dangerous, by supplying two other motivations for dumping there. Previous underestimation of danger (A) does not weaken and could support the argument for implicitly admitting danger. The passage does not identify the area as rural (B). (C) contradicts itself, making no sense. The passage does not identify the area as urban (E).
3. A: The passage states China must raise its GNP by 33% to avoid economic collapse; if 33% is reached, 50% is possible. If 50% is impossible, 33% was not reached; the economy will collapse. China's economy will fall without GNP increase, not vice versa (B); and by 33%, not 50% (B), (C). National conflict is not mentioned; neither is 17% (D) or 71% GNP increase (E).
4. B: If every fish Estelle caught was a salmon, all she caught were salmon. Salmon were not necessarily the only fish she saw (A); she could have seen but not caught other fish. Likewise there were not necessarily no other fish there (C); she just didn't catch them. Estelle caught all the salmon she saw, not all the fish she saw (D). (E) is the same as (A)
5. C: The range of compromises between destroying the country and restricting protestors is ignored. Some protestors do not want to destroy the country (A), but some may. Therefore, emphasis on protesting's importance does not necessarily weaken the conclusion (B). Defining reasons for protesting (D) is not required to support the conclusion, which is based on protestors destroying the country regardless of the issues. That protestors are truly threats (E) strengthens, not weakens, the conclusion.
6. A: More diners eating together are more likely to notice all subsequently experiencing food poisoning. Customers expecting food poisoning would not eat at fast food restaurants (B). Even if expecting more chance of it, they would still report occurrences. More people choosing fast food restaurants (C) would increase the probability of food poisoning complaints from fast food, not family dining, restaurants. Choice (D) simply contradicts (A). Microwaving (E) is not associated with food poisoning.
7. B: Although hopeful growers believe climate changes "should not" change rainfall, these cause annual agricultural yield fluctuations. This claim proceeds from the assumption that since yields already fluctuate from climate change, additional rainfall change would exacerbate fluctuations. That climate change cannot be accurately predicted (A) is not claimed. Technology's significant influence despite climate change makes improved yields likely, NOT "highly unlikely" (C). That rainfall is less predictable than temperature (D), or cool temperatures more threatening (E), are not claims.
8. C: Scientists can observe and study various life stages of breeds too long-lived to study their full life span during scientists' lifetimes. The passage indicates subjects' using the same observational methods, but using the same ideals (A) cannot be assumed. Choice (B) is true, but not indicated in the passage. Today's techniques preventing studying many breeds in the environment (D) is nowhere suggested. Various scientific techniques are available (E), but this cannot be inferred from the passage.
9. C: He believes neither child can take control of his company because of his opinion that they "lack common sense." Therefore, he believes a person needs common sense to run a company. That his children are "ignorant to the experience of controlling the company" (A) is not indicated as his opinion. Nothing is mentioned about obtaining an MBA (B). He thinks both children lack common sense, not one (D). Committee assistance (E) is never mentioned.
10. A: The argument identifies the loophole of unlimited special procedure expenses, and unlimited repeat visit charges for the same problem. It does not imply dental care cannot be regulated (B). Substituting specialized procedures for less-expensive procedures (C) is never suggested. The passage never suggests that regular visits cost more than specialized procedures (D), or that patients cannot distinguish appropriate expenses (E).
11. E: Champions "have all changed to" Wilson's new rackets; they did not win past championships with them. Champions are knowledgeable about tennis and their equipment (A): the ad uses their choosing Wilson's as an example to follow. It indicates exclusive use of oak in Wilson's new line (B). Durability and strength are named as oak's benefits; from the suggestion these will "improve your tennis swing," we can infer "powerful swings" (C) and "you will improve your tennis playing" (D).
12. A: To know one will feel pain from a blood test, we must assume needles always cause this patient pain. Having trouble finding the vein (B) is not the only cause of needle pain. Experiencing pain with this doctor before (C) does not guarantee s/he always will. Leaving a bruise (D) does not always mean the needle hurt. Having to try different needles (E) is not the only cause of pain.
13. B: The student's real-life experience is unlikely to show in writing about fictional characters. Different writing techniques' variable success among writers (A) is not criticism of her choice. Author permission (C) is irrelevant to her choice's inappropriateness. Self-examination is important to originality (D), but misses the point: relevance to students' everyday lives, not originality, was assigned. Instructive, significant themes' precedence over fame (E) is irrelevant to her choosing fictional characters to describe her real-life experiences.
14. C: Monopoly means declining/ therefore, lacking competition. By recognizing its field is competitive, the company accepts competition and has a non-monopolistic focus. Monopoly's definition as a one-seller market (A) is not implied: the statement indicates only that monopoly is "distinguished through the decline or lack of competition." Family competition (B), service industry (D), and public ownership (E) are never implied in the statement.
15. A: If society believes local AA facilities are unsuccessful, society will not fund them. Cost exceeding local funding (B) does not weaken the argument: "society" does not necessarily mean local (and more likely means federal funding.) Neighborhood group support (C) would strengthen, not weaken, the argument for local facilities. Maintenance cost (D) strengthens the argument that if society wants to force facilities on communities, society should pay for them. Alcoholics' treatment resistance (E) is irrelevant to forcing/funding facilities.
16. C: The official attributed Americans' perception of Japanese as "inscrutable" to American lack of discernment, not Japanese inaccessibility. He never suggested all people are inscrutable (A). He did not say that most Americans don't understand Japanese culture specifically (B), but that Americans lack understanding generally. He never said the Japanese mistrust American ambassadors (D). For the offensive "inscrutable" stereotype, he returned a reciprocal opinion of Americans as unperceptive, rather than calling for improved cultural understanding (E).
17. E: The official's response and attitude were compassionate, indicated by his "calm and friendly" look and "gentle voice." His words also demonstrated a sense of humor. The description of his behavior is not fearful (A), emotional (B), angry (C), or indifferent (D). While he deflected a racial stereotype of the Japanese by introducing a corresponding one of Americans, he is not described as doing so with fear, emotion, anger, or indifference.
18. E: The poet humorously couches the fact that he forgot what he meant by saying only he and God knew at the time, and now only God knows-i.e., now he himself does not know. He is not saying God is wiser (A), but remembers better. He never says people can't understand poetry (B); he refers only to himself not knowing/remembering his meaning. He refers only to the verse's meaning, not its inspiration (C), (D).
19. B: This is a syllogism. Syllogisms have three parts: major premise, minor premise, conclusion. Syllogism uses deductive reasoning, reducing general information (all birds have beaks, all sparrows are birds) to infer a more specific conclusion (all sparrows have beaks). Inductive reasoning (D), the opposite of deductive, accumulates specific facts to form generalizations (A). No special training (C) was required to make this conclusion. The argument involves no ambiguity (E): it is not open to multiple interpretations or unclear.
20. E: The ad assumes some people find imported spring water a status symbol by mentioning their embarrassment over serving domestic. It assumes it is hard to tell these apart by flavor (A): "You'll never taste the difference". There is no assumption most spring water is bottled at its source (B). The only reason the ad assumes for higher prices is status, not regulations (C). The decanter conceals water's domestic origin rather than improving its taste (D).
21. B: The paradox is the priest using the parishioner's answer as a criterion for judging itself! He did not later change his accusation (A). Following/violating religious law (C) is irrelevant. There is no reason to think the priest doesn't believe in the devil (D). The questioner rejecting the answer (E) is not necessarily paradoxical if he doesn't like the answer. (This reflects the familiar conundrum: "Are you lying?" "Yes." "Then you must be telling the truth").
2. A pesticide producing company states that their unused pesticide that gets dumped does not pose a threat to the aquatic life in the surrounding area. If this is correct, then why have local fish been dying in this region? Due to the fact that the pesticide company is not located in a highly fish-populated area, they implicitly admit that the pesticides they produce are relatively dangerous to the nearby aquatic life.
7. We are well aware that there are warning signs concerning massive climate changes, and that these climate changes are reducing plant life. Many hopeful crop growers believe that there will not be an overall negative effect on the plant growth population due to the fact that rainfall should not be altered because of the climate changes. However, for the average plant, it is because of the climate change that agricultural technology has an overall yield in annual fluctuation.
8. Different life forms such as animals and plants are known to have certain breeds that have extended life spans compared to that of humans. Due to this fact, scientists pass away before they are able to study the complete life cycle of these certain breeds. This being the case, a single breed may be inferred by observation over various life stages. Geology or astronomy can be applied to using the same method. Scientists can also use this method to learn about desert evolution and rock formations.
10. The regulation that is proposed for dental insurance will contribute only small assistance to patients in the reduction of costs for regular dental care. Although the bill limits the amount that the dentist can charge for a regular visit, it does not limit the expense amount that they can charge if a patient is in need of a special procedure, and it doesn't place any limits on the amount of times that the dentist may see a patient for the same occurring problem. This being the case, instead of the patient being charged once, the dentist can bill the patient numerous times, and the total costs will not reduce.
1. C: If e-book subscriptions remained stable in the five preceding years while textbook purchases declined, this would support the explanation for textbook price increases in response to lower textbook sales. A publication backlog (A) would not raise prices. Student numbers declining (B) does not justify raising textbook prices when e-book sales did not decline. Electronic publishers would not reduce textbook subscriptions (D), since they don't publish these. More available textbooks (E) should lower, not raise, prices.
2. D: The statement that dumping in lower-fish areas is less regulated and less expensive weakens the argument that dumping there implies the dumped pesticide is dangerous, by supplying two other motivations for dumping there. Previous underestimation of danger (A) does not weaken and could support the argument for implicitly admitting danger. The passage does not identify the area as rural (B). (C) contradicts itself, making no sense. The passage does not identify the area as urban (E).
5. C: The range of compromises between destroying the country and restricting protestors is ignored. Some protestors do not want to destroy the country (A), but some may. Therefore, emphasis on protesting's importance does not necessarily weaken the conclusion (B). Defining reasons for protesting (D) is not required to support the conclusion, which is based on protestors destroying the country regardless of the issues. That protestors are truly threats (E) strengthens, not weakens, the conclusion.
6. A: More diners eating together are more likely to notice all subsequently experiencing food poisoning. Customers expecting food poisoning would not eat at fast food restaurants (B). Even if expecting more chance of it, they would still report occurrences. More people choosing fast food restaurants (C) would increase the probability of food poisoning complaints from fast food, not family dining, restaurants. Choice (D) simply contradicts (A). Microwaving (E) is not associated with food poisoning.
7. B: Although hopeful growers believe climate changes "should not" change rainfall, these cause annual agricultural yield fluctuations. This claim proceeds from the assumption that since yields already fluctuate from climate change, additional rainfall change would exacerbate fluctuations. That climate change cannot be accurately predicted (A) is not claimed. Technology's significant influence despite climate change makes improved yields likely, NOT "highly unlikely" (C). That rainfall is less predictable than temperature (D), or cool temperatures more threatening (E), are not claims.
8. C: Scientists can observe and study various life stages of breeds too long-lived to study their full life span during scientists' lifetimes. The passage indicates subjects' using the same observational methods, but using the same ideals (A) cannot be assumed. Choice (B) is true, but not indicated in the passage. Today's techniques preventing studying many breeds in the environment (D) is nowhere suggested. Various scientific techniques are available (E), but this cannot be inferred from the passage.
9. C: He believes neither child can take control of his company because of his opinion that they "lack common sense." Therefore, he believes a person needs common sense to run a company. That his children are "ignorant to the experience of controlling the company" (A) is not indicated as his opinion. Nothing is mentioned about obtaining an MBA (B). He thinks both children lack common sense, not one (D). Committee assistance (E) is never mentioned.
10. A: The argument identifies the loophole of unlimited special procedure expenses, and unlimited repeat visit charges for the same problem. It does not imply dental care cannot be regulated (B). Substituting specialized procedures for less-expensive procedures (C) is never suggested. The passage never suggests that regular visits cost more than specialized procedures (D), or that patients cannot distinguish appropriate expenses (E).
11. E: Champions "have all changed to" Wilson's new rackets; they did not win past championships with them. Champions are knowledgeable about tennis and their equipment (A): the ad uses their choosing Wilson's as an example to follow. It indicates exclusive use of oak in Wilson's new line (B). Durability and strength are named as oak's benefits; from the suggestion these will "improve your tennis swing," we can infer "powerful swings" (C) and "you will improve your tennis playing" (D).
13. B: The student's real-life experience is unlikely to show in writing about fictional characters. Different writing techniques' variable success among writers (A) is not criticism of her choice. Author permission (C) is irrelevant to her choice's inappropriateness. Self-examination is important to originality (D), but misses the point: relevance to students' everyday lives, not originality, was assigned. Instructive, significant themes' precedence over fame (E) is irrelevant to her choosing fictional characters to describe her real-life experiences.
14. C: Monopoly means declining/ therefore, lacking competition. By recognizing its field is competitive, the company accepts competition and has a non-monopolistic focus. Monopoly's definition as a one-seller market (A) is not implied: the statement indicates only that monopoly is "distinguished through the decline or lack of competition." Family competition (B), service industry (D), and public ownership (E) are never implied in the statement.
15. A: If society believes local AA facilities are unsuccessful, society will not fund them. Cost exceeding local funding (B) does not weaken the argument: "society" does not necessarily mean local (and more likely means federal funding.) Neighborhood group support (C) would strengthen, not weaken, the argument for local facilities. Maintenance cost (D) strengthens the argument that if society wants to force facilities on communities, society should pay for them. Alcoholics' treatment resistance (E) is irrelevant to forcing/funding facilities.
16. C: The official attributed Americans' perception of Japanese as "inscrutable" to American lack of discernment, not Japanese inaccessibility. He never suggested all people are inscrutable (A). He did not say that most Americans don't understand Japanese culture specifically (B), but that Americans lack understanding generally. He never said the Japanese mistrust American ambassadors (D). For the offensive "inscrutable" stereotype, he returned a reciprocal opinion of Americans as unperceptive, rather than calling for improved cultural understanding (E).
17. E: The official's response and attitude were compassionate, indicated by his "calm and friendly" look and "gentle voice." His words also demonstrated a sense of humor. The description of his behavior is not fearful (A), emotional (B), angry (C), or indifferent (D). While he deflected a racial stereotype of the Japanese by introducing a corresponding one of Americans, he is not described as doing so with fear, emotion, anger, or indifference.
18. E: The poet humorously couches the fact that he forgot what he meant by saying only he and God knew at the time, and now only God knows-i.e., now he himself does not know. He is not saying God is wiser (A), but remembers better. He never says people can't understand poetry (B); he refers only to himself not knowing/remembering his meaning. He refers only to the verse's meaning, not its inspiration (C), (D).
19. B: This is a syllogism. Syllogisms have three parts: major premise, minor premise, conclusion. Syllogism uses deductive reasoning, reducing general information (all birds have beaks, all sparrows are birds) to infer a more specific conclusion (all sparrows have beaks). Inductive reasoning (D), the opposite of deductive, accumulates specific facts to form generalizations (A). No special training (C) was required to make this conclusion. The argument involves no ambiguity (E): it is not open to multiple interpretations or unclear.
20. E: The ad assumes some people find imported spring water a status symbol by mentioning their embarrassment over serving domestic. It assumes it is hard to tell these apart by flavor (A): "You'll never taste the difference". There is no assumption most spring water is bottled at its source (B). The only reason the ad assumes for higher prices is status, not regulations (C). The decanter conceals water's domestic origin rather than improving its taste (D).
21. B: The paradox is the priest using the parishioner's answer as a criterion for judging itself! He did not later change his accusation (A). Following/violating religious law (C) is irrelevant. There is no reason to think the priest doesn't believe in the devil (D). The questioner rejecting the answer (E) is not necessarily paradoxical if he doesn't like the answer. (This reflects the familiar conundrum: "Are you lying?" "Yes." "Then you must be telling the truth").
8. Different life forms such as animals and plants are known to have certain breeds that have extended life spans compared to that of humans. Due to this fact, scientists pass away before they are able to study the complete life cycle of these certain breeds. This being the case, a single breed may be inferred by observation over various life stages. Geology or astronomy can be applied to using the same method. Scientists can also use this method to learn about desert evolution and rock formations.
10. The regulation that is proposed for dental insurance will contribute only small assistance to patients in the reduction of costs for regular dental care. Although the bill limits the amount that the dentist can charge for a regular visit, it does not limit the expense amount that they can charge if a patient is in need of a special procedure, and it doesn't place any limits on the amount of times that the dentist may see a patient for the same occurring problem. This being the case, instead of the patient being charged once, the dentist can bill the patient numerous times, and the total costs will not reduce.
1. C: If e-book subscriptions remained stable in the five preceding years while textbook purchases declined, this would support the explanation for textbook price increases in response to lower textbook sales. A publication backlog (A) would not raise prices. Student numbers declining (B) does not justify raising textbook prices when e-book sales did not decline. Electronic publishers would not reduce textbook subscriptions (D), since they don't publish these. More available textbooks (E) should lower, not raise, prices.
7. B: Although hopeful growers believe climate changes "should not" change rainfall, these cause annual agricultural yield fluctuations. This claim proceeds from the assumption that since yields already fluctuate from climate change, additional rainfall change would exacerbate fluctuations. That climate change cannot be accurately predicted (A) is not claimed. Technology's significant influence despite climate change makes improved yields likely, NOT "highly unlikely" (C). That rainfall is less predictable than temperature (D), or cool temperatures more threatening (E), are not claims.
13. B: The student's real-life experience is unlikely to show in writing about fictional characters. Different writing techniques' variable success among writers (A) is not criticism of her choice. Author permission (C) is irrelevant to her choice's inappropriateness. Self-examination is important to originality (D), but misses the point: relevance to students' everyday lives, not originality, was assigned. Instructive, significant themes' precedence over fame (E) is irrelevant to her choosing fictional characters to describe her real-life experiences.
15. A: If society believes local AA facilities are unsuccessful, society will not fund them. Cost exceeding local funding (B) does not weaken the argument: "society" does not necessarily mean local (and more likely means federal funding.) Neighborhood group support (C) would strengthen, not weaken, the argument for local facilities. Maintenance cost (D) strengthens the argument that if society wants to force facilities on communities, society should pay for them. Alcoholics' treatment resistance (E) is irrelevant to forcing/funding facilities.
16. C: The official attributed Americans' perception of Japanese as "inscrutable" to American lack of discernment, not Japanese inaccessibility. He never suggested all people are inscrutable (A). He did not say that most Americans don't understand Japanese culture specifically (B), but that Americans lack understanding generally. He never said the Japanese mistrust American ambassadors (D). For the offensive "inscrutable" stereotype, he returned a reciprocal opinion of Americans as unperceptive, rather than calling for improved cultural understanding (E).
19. B: This is a syllogism. Syllogisms have three parts: major premise, minor premise, conclusion. Syllogism uses deductive reasoning, reducing general information (all birds have beaks, all sparrows are birds) to infer a more specific conclusion (all sparrows have beaks). Inductive reasoning (D), the opposite of deductive, accumulates specific facts to form generalizations (A). No special training (C) was required to make this conclusion. The argument involves no ambiguity (E): it is not open to multiple interpretations or unclear.
10. The regulation that is proposed for dental insurance will contribute only small assistance to patients in the reduction of costs for regular dental care. Although the bill limits the amount that the dentist can charge for a regular visit, it does not limit the expense amount that they can charge if a patient is in need of a special procedure, and it doesn't place any limits on the amount of times that the dentist may see a patient for the same occurring problem. This being the case, instead of the patient being charged once, the dentist can bill the patient numerous times, and the total costs will not reduce.

You might also like